1
$\begingroup$

This question is related to the quantization of a massless vector field using the Gupta-Bleuler quantization.

Consider a vector field$A^\mu(x)$,$x\equiv x^\mu$. The field$A^\mu$ transforms according to a gauge transformation:$A_\mu\rightarrow A_\mu^{'}\equiv A_\mu-\partial_\mu\alpha$, where$\alpha$ is some function. Consider the following Lagrangian

$$\mathcal{L}=-\frac{1}{4}F_{\mu\nu}F^{\mu\nu},\quad F_{\mu\nu}\equiv \partial_\mu A_\nu-\partial_\nu A_\mu.$$

This Lagrangian is in particular invariant under the gauge transformation of the field$A^\mu$. The quantization à la Gupta-Bleuler of the field$A^\mu$ consists in adding a term in the Lagrangian that breaks the gauge invariance; for a free parameter$\xi$ one has

$$\mathcal{L}_{GB}=-\frac{1}{4}F_{\mu\nu}F^{\mu\nu}-\frac{\xi}{2}(\partial_\mu A^\mu)^2.$$

The equations of motion of this second Lagrangian do not correspond to the ones coming from the first Lagrangian, which are the Maxwell equations. Given the Fock space$\mathcal{F}$ arrising from the Gupta-Bleuler quantization, one defines the physical states$|\psi\rangle$ as those satisfing$\langle\psi|\partial_\mu A^\mu|\psi\rangle=0$. Equivalently, this condition can be rewritten as

$$\partial_\mu A^\mu=\partial_\mu A_+^\mu+\partial_\mu A_-^\mu,\quad \partial_\mu A_-^\mu=\int \mathrm{d}\Omega_{\vec{k}}(-ik_\mu)a^\mu(\vec{k})e^{-ikx}\Rightarrow \partial_\mu A^\mu_-|\psi\rangle=0.$$

The original Lagrangian is such that the vector field$A^\mu$ has only two independent polarizations;$A^\mu$ has only two degrees of freedom. For the Gupta-Bleuler quantization, I would expect this to hold as well, otherwise we would be describing a different theory.

My question: is it correct that the Gupta-Bleuler quantization also leads to only two independent polarizations of the field$A^\mu$, and how can one show it?

askedJun 22 at 12:34
QuantizedObject's user avatar
$\endgroup$

1 Answer1

0
$\begingroup$

Consider the vector field$A^\mu(\vec{x},t)$, which is quantized à la Gupta-Bleuler. It can be expanded in plane waves as

$$A^\mu(\vec{x},t)=\int \mathrm{d}\Omega_\vec{k}\left(a^\mu(\vec{k})e^{-ikx}+a^{\mu\dagger}(\vec{k})e^{ikx}\right),\quad \mathrm{d}\Omega_\vec{k}=\frac{\mathrm{d}^3 k}{(2\pi)^3 2k_0}.$$

The condition$\partial_\mu A^\mu_-|\Psi\rangle=0$ for a physical state can be rewritten as follows; define first$L(\vec{k})=k_\mu a^\mu(\vec{k})$. A physical state of the Fock space can be defined as

$$\partial_\mu A^\mu_-|\Psi\rangle=0\quad\Leftrightarrow\quad L(\vec{k})|\Psi\rangle=0.$$

Consider now a single particle state given by

$$|\Psi\rangle=\int\mathrm{d}\Omega_\vec{k}f(\vec{k})\varepsilon(\vec{k})^\mu a_\mu^\dagger(\vec{k})|0\rangle,$$

where$f(\vec{k})$ is some function, and$\varepsilon^\mu(\vec{k})$ the polarization of the massless vector field. Using the relation$[a^\mu(\vec{k}),a^{\nu\dagger}(\vec{q})]=-\eta^{\mu\nu}\delta^3(\vec{k}-\vec{q})(2\pi)^3 2q_0$ between the ladder operators of the quantized massless vector field and the definition of a single particle state as above, we obtain that the relation$L(\vec{k})|\Psi\rangle=0$ implies

$$L(\vec{k})|\Psi\rangle=\int \mathrm{d}\Omega_\vec{q}f(\vec{q})k_\mu a^\mu(\vec{k})\varepsilon_\nu(\vec{q}) a^{\nu\dagger}(\vec{q})|0\rangle=-\int\mathrm{d}\Omega_\vec{q}f(\vec{q})2q_0(2\pi)^3\delta^3(\vec{k}-\vec{q})k_\mu \varepsilon^\mu(\vec{q})|0\rangle=0.$$

This last relation implies the condition$k_\mu\varepsilon^\mu(\vec{k})=0$ on the polarization, which reduces its degrees of freedom to$3$ (instead of the four of a generic four-vector object$O^\mu)$. We can further reduces the degrees of freedom of the polarization vector by noting that the polarization vector, as it satisifes$k_\mu\varepsilon^\mu(\vec{k})$, can be rewritten as$\varepsilon_\mu=\varepsilon_\mu^\perp+a_Lk_\mu$, where$\varepsilon_\mu^\perp=c_1\varepsilon_\mu^1+c_2\varepsilon^2_\mu$,$1,2$ are the directions transverse w.r.t.$\vec{k}$. Note for this to hold we require that$k_\mu k^\mu=0$, which is the case for a massless vector field. This relation implies that$k^\mu\varepsilon_\mu^\perp=0$, and that$\varepsilon_\mu^\perp$ has zero time component (by its definition of being transvere w.r.t.$\vec{k}$). Observe how$\varepsilon^\perp_\mu$ only has two degrees of freedom.

Using the above relations, we can compute the norm of a one particle state

$$\langle\Psi|\Psi\rangle=\int \mathrm{d}\Omega_\vec{p} \mathrm{d}\Omega_\vec{k}f^*(\vec{p})f(\vec{k})\varepsilon^{\mu *}(\vec{p})\varepsilon(\vec{k})\langle0 |a_\mu(\vec{p})a_\nu^\dagger(\vec{k})|0\rangle=\int \mathrm{d}\Omega_\vec{p}|\vec{\varepsilon}^\perp(\vec{p})|^2\geq 0,$$

where I used the commutation relation between the ladder operators, and that$\varepsilon^\perp_\mu$ has zero time component. This shows that only the element$\varepsilon^\perp_\mu$ affect the norm. As such, only two degrees of freedom are relevant when computing the norm of a physical state, which is the result one obtains from the quantization of the Lagrangian$\mathcal{L}=-\frac{1}{4} F_{\mu\nu}F^{\mu\nu}$, as wanted.

answeredJul 3 at 13:51
QuantizedObject's user avatar
$\endgroup$

Your Answer

Sign up orlog in

Sign up using Google
Sign up using Email and Password

Post as a guest

Required, but never shown

By clicking “Post Your Answer”, you agree to ourterms of service and acknowledge you have read ourprivacy policy.

Start asking to get answers

Find the answer to your question by asking.

Ask question

Explore related questions

See similar questions with these tags.